LSAT and Law School Admissions Forum

Get expert LSAT preparation and law school admissions advice from PowerScore Test Preparation.

 Administrator
PowerScore Staff
  • PowerScore Staff
  • Posts: 8950
  • Joined: Feb 02, 2011
|
#72654
Complete Question Explanation

Strengthen—Principle, SN. The correct answer choice is (E).

Answer choice (A):

Answer choice (B):

Answer choice (C):

Answer choice (D):

Answer choice (E): This is the correct answer choice.


This explanation is still in progress. Please post any questions below!
User avatar
 nzLSAT
  • Posts: 21
  • Joined: Jul 03, 2021
|
#90949
Why is this answer choice E and not C?
 Jeremy Press
PowerScore Staff
  • PowerScore Staff
  • Posts: 1000
  • Joined: Jun 12, 2017
|
#91003
Hi nz,

A good prephrase will definitively eliminate answer choice C on this question. The ideal principle to strengthen any argument is one that ties the premises to the conclusion conditionally. But you have to make sure that the conditional connection "flows" in the right direction. It needs to be [Premises] :arrow: [Conclusion]. In other words, the premise information should be put into the sufficient condition, and the conclusion information should be put into the necessary condition.

Prephrasing in this question would give us this: If electing judges rather than appointing them would be likely to produce conflicts of interest, then judges should be appointed. Simplified: Likely Conflicts of Interest :arrow: Appointed.

Answer choice C reverses this: it puts the "appointing" in the sufficient condition, and the "conflicts of interest" in the necessary condition. It's the Mistaken Reversal of what we need (though even if you reversed the conditionality of C you'd still have some problems; which means it's actually AT BEST the Mistaken Reversal of what we need). The Mistaken Reversal principle will never be the best answer in these questions.

Answer choice E is much better because it matches the conditional flow we're looking for. The conditional structure of E (rephrased) is this: If a public office for which election campaigning would be likely to produce conflicts of interest, then it should NOT be changed from appointed to elected (meaning it should be appointed).
Simplified: Likely Conflicts of Interest :arrow: Appointed. That's our prephrase!

I hope this helps!
User avatar
 BeyondPossum
  • Posts: 4
  • Joined: Jun 26, 2024
|
#107281
I want to point out that this question has a flat-out stylistic error and inconsistency. It starts by referencing offices with "elected office", but then randomly switches to the nonstandard locution "elective office" in one of the options.
 Luke Haqq
PowerScore Staff
  • PowerScore Staff
  • Posts: 927
  • Joined: Apr 26, 2012
|
#107350
Hi BeyondPossum!

I can see that answer choice (B) mentions "elective office." I'm not seeing any mention of "elected office" specifically in the stimulus, however.

I'm not entirely sure why they chose "elective" for (B), and it's not entirely clear if they meant for it to have a different meaning than "elected." If that word choice made you select (B) or if you were inclined to select it, feel free to explain your reasoning more and we'd be happy to address that answer choice further.

Get the most out of your LSAT Prep Plus subscription.

Analyze and track your performance with our Testing and Analytics Package.